luận văn thạc sỹ toán học đồng nhất thức và bất đẳng thức trong tam giác

62 471 1
luận văn thạc sỹ toán học đồng nhất thức và bất đẳng thức trong tam giác

Đang tải... (xem toàn văn)

Tài liệu hạn chế xem trước, để xem đầy đủ mời bạn chọn Tải xuống

Thông tin tài liệu

ĐẠI HỌC THÁI NGUYÊN TRƯỜNG ĐẠI HỌC KHOA HỌC Nguyễn Thị Út ĐỒNG NHẤT THỨC VÀ BẤT ĐẲNG THỨC TRONG TAM GIÁC SOME IDENTITIES AND INEQUALITIES OF TRIANGLES Chuyên ngành: PHƯƠNG PHÁP TOÁN SƠ CẤP Mã số: 60.46.40 LUẬN VĂN THẠC SĨ TOÁN HỌC Người hướng dẫn khoa học: PGS.TS. Đàm Văn Nhỉ Thái Nguyên - 2012 Công trình được hoàn thành tại Trường Đại học Khoa học - Đại học Thái Nguyên Người hướng dẫn khoa học: PGS.TS. Đàm Văn Nhỉ Phản biện 1: Phản biện 2: Luận văn sẽ được bảo vệ trước hội đồng chấm luận văn họp tại: Trường Đại học Khoa học - Đại học Thái Nguyên Ngày tháng năm 2012 Có thể tìm hiểu tại Thư viện Đại học Thái Nguyên 1 Mục lục Mục lục . . . . . . . . . . . . . . . . . . . . . . . . . . . . . 1 Mở đầu . . . . . . . . . . . . . . . . . . . . . . . . . . . . . . 2 Chương 1. Kiến thức chuẩn bị 4 1.1. Bất đẳng thức qua tam thức bậc hai . . . . . . . . . . . 4 1.2. Bất đẳng thức Jensen . . . . . . . . . . . . . . . . . . . . 5 1.3. Bất đẳng thức Karamata, Schur, Muirheard . . . . . . . 8 1.4. Một vài hàm tự chọn . . . . . . . . . . . . . . . . . . . . 12 Chương 2. Một số đồng nhất thức và bất đẳng thức trong tam giác 14 2.1. Đa thức bậc ba liên quan đến tam giác. . . . . . . . . . 14 2.2. Một số bất đẳng thức trong tam giác . . . . . . . . . . . 25 2.3. Một số bài toán nhận dạng tam giác . . . . . . . . . . . 37 Chương 3. Trình bày một số kết quả của J.Liu [8] và của Klamkin [7]. 43 3.1. Khai thác bài toán véc tơ trong mặt phẳng . . . . . . . . 43 3.2. Trình bày lại kết quả bài báo của J.Liu . . . . . . . . . . 51 3.2.1. Một số định lý . . . . . . . . . . . . . . . . . . . 51 3.2.2. Một vài bổ đề . . . . . . . . . . . . . . . . . . . . 52 3.2.3. Chứng minh ba định lý trên . . . . . . . . . . . . 53 3.3. Trình bày bất đẳng thức của Klamkin . . . . . . . . . . 55 Kết luận . . . . . . . . . . . . . . . . . . . . . . . . . . . . . 59 Tài liệu tham khảo . . . . . . . . . . . . . . . . . . . . . . . 60 2 Mở đầu Đồng nhất thức và bất đẳng thức trong tam giác là một chuyên mục hấp dẫn đối với những người quan tâm tới Hình sơ cấp. Đây là một mảnh đất đã được cày xới quá nhiều qua năm tháng. Vấn đề đặt ra: Làm thế nào để có đồng nhất thức và bất đẳng thức mới trong tam giác. Tam giác là một hình quen thuộc đối với tất cả mọi người. Thông thường, khi xét bài toán hình học người ta thường phải dùng đến thước kẻ, compa và giải quyết bài toán ấy qua hình vẽ. Nhưng cách làm như vậy rất khó phát hiện ra hệ thức mới. Chúng ta càng khó xây dựng được bài toán với nhiều đại lượng của tam giác. Do có quá nhiều kết quả trong tam giác nên xuất hiện câu hỏi thứ nhất: Có thể xây dựng được kết quả mới hay không? Nhiều người sử dụng lượng giác, hình vẽ, phương pháp diện tích,v.v để tạo ra kết quả. Theo chúng tôi, những cách xây dựng như vậy rất khó đưa ra hệ thức cho tam giác mà có nhiều thành phần tham gia. Rất tự nhiên, xuất hiện câu hỏi thứ hai: Xây dựng kết quả như thế nào? Bài toán đặt ra: Xây dựng đồng nhất thức và bất đẳng thức trong tam giác. Với luận văn này, chúng tôi mong muốn giải quyết được một phần nào đó thuộc bài toán trên. Luận văn được chia ra làm ba chương. Chương 1. Kiến thức chuẩn bị. Chương này tập trung trình bày về một số bất đẳng thức. Nó bao gồm các mục: Bất đẳng thức qua tam thức bậc 2, bất đẳng thức Jensen qua hàm lồi và bất đẳng thức Muirheard, Karamata. Ngoài ra, để phát hiện ra một số bất đẳng thức khác nữa cho tam giác chúng tôi đã chọn ra một số hàm tương ứng với mục đích đặt ra. Chương 2. Một số đồng nhất thức và bất đẳng thức trong tam giác. Đây là nội dung trọng tâm của luận văn. Nó bao gồm các mục sau: Mục 3 2.1 tập trung xây dựng một số đa thức bậc ba liên quan tam giác. Từ những đa thức này ta đã có thể phát hiện ra một số đồng nhất thức và bất đẳng thức mới trong tam giác. Mục 2.2 tập trung xây dựng và chứng minh lại một số bất đẳng thức trong tam giác qua việc sử dụng các kết quả ở Chương 1. Từ các kết quả đạt được chúng ta sẽ phát hiện ra những tam giác đặc biệt với điều kiện ban đầu đặt ra ở Mục 2.3. Chương 3. Trình bày một số kết quả của J.Liu [8] và của Klamkin [7]. Chương này dành để trình bày việc khai thác một bài toán véc tơ trong mặt phẳng ở Mục 3.1. Mục 3.2 trình bày lại một số kết quả của J. Liu trong bài báo [8]. Mục 3.3 trình bày lại kết quả của Klamkin trong [7]. Luận văn này được hoàn thành với sự hướng dẫn và chỉ bảo tận tình của PGS,TS Đàm Văn Nhỉ. Từ đáy lòng mình, em xin được bày tỏ lòng biết ơn sâu sắc đối với sự quan tâm, động viên và sự chỉ bảo hướng dẫn của thầy. Em xin trân trọng cảm ơn tới các Thầy, Cô giáo trong Trường Đại học Khoa học - Đại học Thái Nguyên, phòng Đào tạo Trường Đại học Khoa học. Đồng thời tác giả xin gửi lời cảm ơn tới tập thể lớp Cao học Toán K4 Trường Đại học Khoa học đã động viên giúp đỡ tôi trong quá trình học tập và làm luận văn này. Tuy nhiên do sự hiểu biết của bản thân, điều kiện thời gian và khuôn khổ của luận văn thạc sĩ, nên chắc rằng trong quá trình nghiên cứu không tránh khỏi những khiếm khuyết.Tác giả rất mong được sự chỉ dạy và đóng góp ý kiến của các Thầy, Cô giáo và quý vị bạn đọc đóng góp ý kiến để luận văn được hoàn thành tốt hơn. Thái Nguyên, ngày 10 tháng 10 năm 2012 Tác giả Nguyễn Thị Út 4 Chương 1 Kiến thức chuẩn bị 1.1. Bất đẳng thức qua tam thức bậc hai Xét tam thức bậc hai f(x) = ax 2 + bx + c, a = 0, ∆ = b 2 −4ac. Ta có các kết quả sau đây: Định lý 1.1.1. f(x) > 0 với ∀x khi và chỉ khi  a > 0 ∆ < 0. Định lý 1.1.2. f(x)  0 với ∀x khi và chỉ khi  a > 0 ∆  0. Định lý 1.1.3. f(x) < 0 với ∀x khi và chỉ khi  a < 0 ∆ < 0. Định lý 1.1.4. f(x)  0 với ∀x khi và chỉ khi  a < 0 ∆  0. Định lý 1.1.5. f(x) = 0 có nghiệm x 1 , x 2 khi và chỉ khi ∆  0. Khi đó: f(x) = a(x −x 1 )(x −x 2 ) và    x 1 + x 2 = −b a x 1 x 2 = c a . Thông thường ta chọn x 1  x 2 . Định lý 1.1.6. x 1 < α < x 2 khi và chỉ khi af(α) < 0. Định lý 1.1.7. α < x 1  x 2 khi và chỉ khi        af(α) > 0 ∆  0 α < −b 2a . Định lý 1.1.8. x 1  x 2 < α khi và chỉ khi        af(α) > 0 ∆  0 α > −b 2a . 5 1.2. Bất đẳng thức Jensen Mục này trình bày Bất đẳng thức Jensen. Nó sẽ được sử dụng để chứng minh một số bất đẳng thức trong tam giác. Trước tiên ta chứng minh bất đẳng thức này. Định nghĩa 1.2.1. Hàm số y = f(x) được gọi là hàm lồi, (xuống phía dưới), trong khoảng (a; b) nếu với mọi a < x 1 , x 2 < b và mọi α ∈ (0; 1) luôn có bất đẳng thức: αf(x 1 ) + (1 − α)f (x 2 )  f  αx 1 + (1 − α)x 2  . Định nghĩa 1.2.2. Hàm số y = f (x) được gọi là hàm lõm, (lên phía trên), trong khoảng (a; b) nếu với mọi a < x 1 , x 2 < b và mọi α ∈ (0; 1) luôn có bất đẳng thức: αf(x 1 ) + (1 − α)f (x 2 )  f  αx 1 + (1 − α)x 2  . Mệnh đề 1.2.1. Giả sử y = f(x) xác định và liên tục trong (a; b) với a < b. Hàm y = f(x) là lồi trong khoảng (a; b) khi và chỉ khi f(x) − f(x 1 ) x − x 1  f(x 2 ) − f(x) x 2 − x hoặc       1 x 1 f(x 1 ) 1 x f(x) 1 x 2 f(x 2 )        0 với mọi x 1 , x, x 2 ∈ (a; b) thỏa mãn x 1 < x < x 2 . Chứng minh. Giả sử y = f(x) là hàm lồi trong khoảng (a; b). Với x 1 , x, x 2 ∈ (a; b), x 1 < x < x 2 , có biểu diễn: x = x 2 − x x 2 − x 1 x 1 + x − x 1 x 2 − x 1 x 2 , f(x)  x 2 − x x 2 − x 1 f(x 1 ) + x − x 1 x 2 − x 1 f(x 2 ). Như vậy có bất đẳng thức (x 2 −x)f(x 1 )+(x 1 −x 2 )f(x)+(x−x 1 )f(x 2 )  0 hay biểu diễn dạng       1 x 1 f(x 1 ) 1 x f(x) 1 x 2 f(x 2 )        0. Điều ngược lại là hiển nhiên. Mệnh đề 1.2.2. Giả sử y = f(x) xác định và liên tục trong khoảng (a; b) và có đạo hàm hữu hạn f  (x). Khi đó y = f(x) là hàm lồi nếu và chỉ nếu f  (x) là hàm không giảm trong (a; b). 6 Chứng minh. Giả sử y = f(x) là hàm lồi trong khoảng (a; b). Với x 1 , x, x 2 ∈ (a; b), x 1 < x < x 2 , có hai biểu diễn sau đây: x = x 2 − x x 2 − x 1 x 1 + x − x 1 x 2 − x 1 x 2 và f(x) − f(x 1 ) x − x 1  f(x 2 ) − f(x) x 2 − x . Khi đó f  (x 1 ) = lim x→x 1 f(x) − f(x 1 ) x − x 1  f(x 2 ) − f(x 1 ) x 2 − x 1  lim x→x 2 f(x 2 ) − f(x) x 2 − x = f  (x 2 ). Như vậyf  (x 1 )  f  (x 2 ). Ngược lại, giả thiết f  (x) là hàm không giảm trong (a; b). Với x 1 , x, x 2 ∈ (a; b), x 1 < x < x 2 ta có f(x) − f(x 1 ) x − x 1 = f  (α) và f(x 2 ) − f(x) x 2 − x = f  (β), trong đó x 1 < α < x < β < x 2 . Vì f  (α)  f  (β) suy ra f(x) − f(x 1 ) x − x 1  f(x 2 ) − f(x) x 2 − x . Vậy y = f(x) là hàm lồi theo Mệnh đề 1.2.1. Từ Mệnh đề 1.2.2 suy ra ngay kết quả dưới đây: Định lý 1.2.1. Giả thiết y = f(x) xác định và liên tục trong khoảng (a; b). Giả sử f(x) có đạo hàm f  (x) cũng liên tục và có f”(x) hữu hạn trong khoảng (a; b). Khi đó y = f(x) là hàm lồi nếu và chỉ nếu f”(x)  0 trong (a; b). Định lý 1.2.2. [Jensen] Nếu y = f (x) là hàm lồi trong khoảng (a; b) thì với mọi a 1 , . . . , a n ∈ (a; b) và mọi số thực α 1 , . . . , α n  0, n  k=1 α k = 1, n  2, ta luôn có bất đẳng thức dưới đây: α 1 f(a 1 )+α 2 f(a 2 )+···+α n f(a n )  f(α 1 a 1 + α 2 a 2 + ··· + α n a n ). Chứng minh. Quy nạp theo n. Với n = 2 kết luận hiển nhiên đúng theo định nghĩa. Giả sử kết luận đã đúng cho n  2. Xét n + 1 điểm a 1 , . . . , a n , a n+1 ∈ (a; b) và các số thực α 1 , . . . , α n , α n+1  0, n+1  k=1 α k = 1 và α n+1 > 0. Đặt b n = α n α n + α n+1 a n + α n+1 α n + α n+1 a n+1 ∈ (a; b). Theo giả thiết quy nạp ta có: f(α 1 a 1 + α 2 a 2 + ··· + α n−1 a n−1 + α n a n + α n+1 a n+1 ) = f(α 1 a 1 + α 2 a 2 + ··· + α n−1 a n−1 + (α n + α n+1 )b n )  α 1 f(a 1 ) + α 2 f(a 2 ) + ···+ α n−1 f(a n−1 ) + (α n + α n+1 )f(b n ). 7 Vì f (b n ) = f ( α n α n + α n+1 a n + α n+1 α n + α n+1 a n+1 )  α n f(a n ) α n + α n+1 + α n+1 f(a n+1 ) α n + α n+1 nên n+1  k=1 α k f(a k )  n+1  k=1 f(α k a k ). Như vậy định lý đã được chứng minh. Chú ý: Đối với các hàm số lõm ta có dấu bất đẳng thức ngược lại. Ví dụ 1.2.1. Giả thiết số nguyên n  2. Chứng minh bất đẳng thức: n  k=1 3 k − 1 3 k−1   3 − 3 2n + 3 2n.3 n  n . Bài giải. Vì f(x) = lnx, x > 0, là hàm lồi nên theo định lý 1.2.2 có 1 n  n  k=1 ln 3 k − 1 3 k−1   ln  1 n  n  k=1 ln 3 k − 1 3 k−1  = ln  3 − 3 2n + 3 2.3 n  . Từ đây suy ra bất đẳng thức n  k=1 3 k − 1 3 k−1   3 − 3 2n + 3 2n.3 n  n . Hệ quả 1.2.1. Với a 1 , . . . , a n , b 1 , . . . , b n , α 1 , . . . , α n > 0, n  k=1 α k = 1 và n  2, ta luôn có các bất đẳng thức dưới đây: (i) n  k=1 α k a k  n  k=1 a α k k . (ii) n  k=1 (a k + b k ) α k  n  k=1 a α k k + n  k=1 b α k k . (iii) m  k=1  m  j=1 a kj  α k  m  j=1 n  k=1 a α k kj với mọi a kj  0. (iv) [Cauchy] n  k=1 a k  n n  n  k=1 a k . Chứng minh. (i) Xét hàm lõm f (x) = ln x. Theo Định lý Jensen ta có ln  n  k=1 a α k k  = n  k=1 α k ln a k  ln  n  k=1 α k a k  . Do n  k=1 a α k k  n  k=1 α k a k . (ii) Do n  k=1 α k a k a k + b k  n  k=1  a k a k + b k  α k , n  k=1 α k b k a k + b k  n  k=1  b k a k + b k  α k theo (i) nên sau khi cộng hai vế được 1  n  k=1  a k a k + b k  α k + 8 n  k=1  b k a k + b k  α k . Qua quy đồng ta nhận được bất đẳng thức n  k=1 (a k + b k ) α k  n  k=1 a α k k + n  k=1 b α k k . (iii) Sử dụng (ii) để quy nạp theo m sẽ được (iii). Với α 1 = ··· = α n = 1 n , từ (i) suy ra (iv). 1.3. Bất đẳng thức Karamata, Schur, Muirheard Bộ n số thực (a) = (a 1 , a 2 , . . . , a n ) thỏa mãn a 1  a 2  ···  a n được gọi là bộ số không tăng. Đặt |(a)| = a 1 + a 2 + ··· + a n . Trong tập hợp tất cả các bộ số không tăng A = {(a) = (a k )} ta định nghĩa quan hệ thứ tự: Giả sử (a) = (a k ), (b) = (b k ) là hai bộ số không tăng. Định nghĩa (a)  (b) khi và chỉ khi a 1 + ··· + a k  b 1 + ··· + b k , ∀ k = 1, 2, . . . , n; Còn nếu có k để a 1 + ··· + a k > b 1 + ··· + b k thì ta viết (a) > (b). Định nghĩa 1.3.1. Giả sử có hai bộ số không tăng (a) = (a 1 , a 2 , . . . , a n ) và (b) = (b 1 , b 2 , . . . , b n ). Bộ (a) được gọi là trội hơn (b) nếu các điều kiện sau đây được thỏa mãn:  a 1 + ··· + a k  b 1 + ··· + b k k = 1, 2, . . . , n −1; |(a)| = |(b)|. Mệnh đề 1.3.1. [Karamata] Giả sử y = f(x) là một hàm lồi trên khoảng (a; b) và các bộ không tăng (a), (b) với a k , b k ∈ (a; b) với k = 1, 2, . . . , n. Nếu bộ (a) trội hơn bộ (b) thì có bất đẳng thức n  k=1 f(a k )  n  k=1 f(b k ); còn khi y = f(x) là hàm lõm thì n  k=1 f(a k )  n  k=1 f(b k ). Chứng minh: Đặt c k = δ f (a k , b k ) = f(b k ) − f(a k ) b k − a k , k = 1, 2, . . . , n. Theo Mệnh đề 1.1.1, dãy (c k ) là dãy đơn điệu giảm bởi vì (a) và (b) là dãy không tăng. Đặt A k = k  i=1 a i , B k = k  i=1 b i với A 0 = B 0 = 0, k = 1, 2, . . . , n. [...]... nhất thức và bất đẳng thức trong tam giác Mục này tập trung trình bày một phương pháp phát hiện ra các đồng nhất thức và bất đẳng thức trong tam giác qua phương trình đa thức bậc ba 2.1 Đa thức bậc ba liên quan đến tam giác Cho ∆ABC với độ dài ba cạnh là a, b, c; bán kính các đường tròn nội, ngoại tiếp là r, R; bán kính đường tròn bàng tiếp là r1 , r2 , r3 ; nửa chu vi p và diện tích S Ta sẽ chỉ ra a,... b3 2 a5 b2 c2 + a2 b5 c2 + a2 b2 c5 Bất đẳng thức này tương đương với bất đẳng thức M(6,3,0) (a) M(5,2,2) (a) Ví dụ 1.3.5 Với ba số thực dương a, b, c, ta luôn có bất đẳng thức: a3 + b3 + c3 (a + b) (b + c) (c + a) Chứng minh Bất đẳng thức cần chứng minh tương đương với abc (a + b − c) (b + c − a) (c + a − b) Nếu vế phải có một hoặc ba thừa số âm thì bất đẳng thức hiển nhiên đúng Nếu vế phải chỉ... , , an , xảy ra bất đẳng thức Mα (a) Mβ (a) khi và chỉ khi α β và |α| = |β| Dấu bằng chỉ xảy ra khi α = β và a1 = a2 = · · · = an Chú ý rằng, khi vận dụng Bất đẳng thức Muirheard ta phải chọn bộ trội thế nào để nhanh có kết quả Ví dụ 1.3.4 Với ba số thực dương a, b, c chúng ta có bất đẳng thức: 1 1 1 + 3 + 3 a3 + b3 + abc b + c3 + abc c + a3 + abc 1 abc Chứng minh Bất đẳng thức tương đương với:... R đẳng thức còn lại 2 2 2m2 a Ví dụ 2.2.12 Với mọi tam giác ABC ta luôn có bất đẳng thức sau đây: ab + bc + ca (i) la + lb + lc 2R (ii) Giả sử I là giao điểm ba đường phân giác trong của ∆ABC và AI, BI, CI cắt đường tròn ngoại tiếp ∆ABC ở A2 , B2 , C2 , tương ứng Khi 31 đó ta có bất đẳng thức AA2 + BB2 + CC2 > a + b + c 2r (iii) AA2 + BB2 + CC2 6R 3 Dấu bằng xảy ra khi ∆ABC đều R √ a 1 1 1 3 b c và. .. (bβ − cβ )(bβ − aβ ) 0, hay aα (aβ − cβ ) − bα (bβ − cβ ) 0, nhưng bất đẳng thức này là hiển nhiên Ví dụ 1.3.3 Giả sử a, b, c là độ dài ba cạnh một tam giác Chứng minh rằng: a3 + b3 + c3 + 3abc a2 b + a2 c + b2 c + b2 a + c2 a + c2 b Bài giải Bất đẳng thức có được từ Mệnh đề 1.3.2 với α = β = 1 Bổ đề 1.3.1 Nếu β = Tij (α) thì có bất đẳng thức Mβ (a) Dấu bằng xảy ra khi a1 = · · · = an Mα (a) Chứng minh... ( 2.2.6 ) và ( 2.2.7 ) 2 √ √ √ nên ta phải chứng minh la +lb +lc s2 − u2 + s2 + 8u2 3 (s + u) √ √ Với u s 2u Có thể coi u = 1 và chỉ ra s2 − 1 + s2 + 8 29 √ 3 (s + 1) với 1 s 2, điều này là hiển nhiên √ 1 la + lb + lc 3 Tóm lại 2 a+b+c 2 Tiếp theo là một số bất đẳng thức giữa các cạnh, đường cao và bán kính đường tròn nội, ngoại tiếp Ví dụ 2.2.9 Với mọi tam giác ABC ta luôn có bất đẳng thức sau đây:... b và c là nghiệm của phương trình ẩn t: t2 −(8−a)t+(a2 −8a+16) = 0 Vì phương trình này có nghiệm nên ∆ = (8 − a)2 − 4(a2 − 8a + 16) 0 16 16 16 hay 0 a Tương tự ta có 0 b và 0 c 3 3 3 Chứng minh Từ giả thiết ta suy ra Hoàn toàn tương tự ta có bài toán sau: Cho a, b, c là ba số thỏa mãn 20 a + b + c = 10 hệ Khi đó 0 a, b, c ab + bc + ca = 25 3 14 Chương 2 Một số đồng nhất thức và bất đẳng thức trong. .. αx Chứng minh Với n = 1 ta có đẳng thức xảy ra Giả sử bất đẳng thức đúng đến n = k 1 ta sẽ đi chứng minh bất đẳng thức đúng đến n = k + 1 Thật vậy ta có (1 + x)k+1 = (1 + x)k (1 + x) (1 + kx) (1 + x) = 1 + kx + x + kx2 1 + (k + 1) x Ví dụ 1.4.1 Chứng minh rằng, nếu α1 , α2 , α3 > 0 và α1 + α2 + α3 = 1 thì với mọi số thực u1 , u2 , u3 0 có bất đẳng thức uα1 uα2 uα3 1 2 3 α1 u1 + α2 u2 + α3 u3 Bài giải... 1.3.2 [Schur] Với α, β > 0 có S(α+2β,0,0) + S(α,β,β) 2S(α+β,β,0) Chứng minh Bất đẳng thức cần chứng minh chính là bất đẳng thức dưới đây: aα+2β + bα+2β + cα+2β + aα bβ cβ + aβ bα cβ) + aβ bβ cα aα+β bβ + aα+β cβ + bα+β aβ + bα+β cβ + cα+β aβ + cα+β bβ , (∗), với a, b, c 0 Không hạn chế có thể giả thiết a b c 0 Viết lại bất đẳng thức (*) thành aα (aβ − bβ )(aβ − cβ ) + bα (bβ − cβ )(bβ − aβ ) + cα (cβ... 8Rr + 2r2 2r2 = ⇔ ha + hb + hc 8r + 8R R R Do bởi R Ví dụ 2.2.11 Với mọi tam giác ABC ta luôn có bất đẳng thức sau đây: (i) 2R (ma + mb + mc ) a2 + b2 + c2 9 (ii) R ma + mb + mc 2 (iii) [VMO 1991] Giả sử G là trọng tâm của ∆ABC và AG, BG, CG cắt đường tròn ngoại tiếp ∆ABC ở A1 , B1 , C1 tương ứng Ta có bất đẳng √ 1 1 1 3 1 1 1 thức: + + 3 + + R GA1 GB1 GC1 a b c a2 Bài giải (i) Gọi M là trung điểm . một số bất đẳng thức. Nó bao gồm các mục: Bất đẳng thức qua tam thức bậc 2, bất đẳng thức Jensen qua hàm lồi và bất đẳng thức Muirheard, Karamata. Ngoài ra, để phát hiện ra một số bất đẳng thức. thức bậc ba liên quan tam giác. Từ những đa thức này ta đã có thể phát hiện ra một số đồng nhất thức và bất đẳng thức mới trong tam giác. Mục 2.2 tập trung xây dựng và chứng minh lại một số bất. ra: Làm thế nào để có đồng nhất thức và bất đẳng thức mới trong tam giác. Tam giác là một hình quen thuộc đối với tất cả mọi người. Thông thường, khi xét bài toán hình học người ta thường phải

Ngày đăng: 09/11/2014, 12:03

Từ khóa liên quan

Tài liệu cùng người dùng

Tài liệu liên quan